Administración     

Olimpiadas de Matemáticas
Página de preparación y problemas

Selector
La base de datos contiene 1154 problemas y 775 soluciones.
OME Local
OME Nacional
OIM
OME Andalucía
Retos UJA
—20
Problema 1086
Dado un par de números reales $(x,y)$ tales que $0\leq x\leq y\leq 1$, sea \[M(x,y)=\max\{xy,1-x-y+xy,x+y-2xy\}.\] Hallar el mínimo valor que puede tomar $M(x,y)$ para todos estos pares $(x,y)$.
Sin pistas
Sin soluciones
info
Si crees que el enunciado contiene un error o imprecisión o bien crees que la información sobre la procedencia del problema es incorrecta, puedes notificarlo usando los siguientes botones:
Informar de error en enunciado Informar de procedencia del problema
Problema 1080
Prueba que, para todo $a,b,c\gt 0$, se cumple que \[\frac{a^2}{b^3c}-\frac{a}{b^2}\geq \frac{c}{b}-\frac{c^2}{a}.\] ¿En qué caso se cumple la igualdad?
pistasolución 1info
Pista. Factoriza la expresión $\frac{a^2}{b^3c}-\frac{a}{b^2}-\frac{c}{b}+\frac{c^2}{a}$ poniendo previamente denominador común.
Solución. Observemos que \begin{align*} \frac{a^2}{b^3c}-\frac{a}{b^2}-\frac{c}{b}+\frac{c^2}{a}&=\frac{a^3-a^2 b c-a b^2 c^2+b^3 c^3}{a b^3 c}\\ &=\frac{c^2}{a}\cdot\left(\frac{a^3}{b^3c^3}-\frac{a^2}{b^2c^2}-\frac{a}{bc}+1\right). \end{align*} Obtenemos así el polinomio $x^3-x^2-x+1$ tras el cambio $x=\frac{a}{bc}$. Este polinomio se puede factorizar como $(x-1)^2(x+1)$, luego podemos proseguir factorizando como \begin{align*} \frac{a^2}{b^3c}-\frac{a}{b^2}-\frac{c}{b}+\frac{c^2}{a} &=\frac{c^2}{a}\cdot\left(\frac{a}{bc}-1\right)^2\left(\frac{a}{bc}+1\right)\geq 0. \end{align*} La igualdad se da cuando el factor $\frac{a}{bc}-1$ se anula (ya que el resto de factores son estrictamente positivos), es decir, cuando $a=bc$.
Si crees que el enunciado contiene un error o imprecisión o bien crees que la información sobre la procedencia del problema es incorrecta, puedes notificarlo usando los siguientes botones:
Informar de error en enunciado Informar de procedencia del problema
Problema 1047
Determinar los números reales $x\gt 1$ para los cuales existe un triángulo cuyos lados tienen longitudes \[x^4+x^3+2x^2+x+1,\qquad 2x^3+x^2+2x+1,\qquad x^4-1.\]
pistasolución 1info
Pista. Determina cuál es el lado mayor y fíjate en que no debe alcanzar a la suma de los otros dos para que exista el triángulo (desigualdad triangular).
Solución. En primer lugar, vamos a determinar cuál es el lado mayor. Por un lado, \[x^4+x^3+2x^2+x+1\gt x^4+1\gt x^4-1,\] ya que $x$ es positivo. Usando que $x\gt 1$, tenemos que $x^4\geq x^3$ y $x^2\gt x$, luego \[x^4+x^3+2x^2+x+1\gt x^3+x^3+x^2+x+x+1=2x^3+x^2+2x+1.\] Sabiendo entonces que el primer lado es el mayor, tendremos que ver cuándo no supera a la suma de los otros dos, es decir, la respuesta al enunciado serán los números $x\gt 1$ tales que \[x^4+x^3+2x^2+x+1\lt (2x^3+x^2+2x+1)+(x^4-1).\] Tras simplificar y factorizar, nos queda $-x^3+x^2-x+1\lt 0$ y podemos factorizar el miembro de la izquierda para llegar a la desgualdad $(x^2+1)(1-x)\lt 0$, desigualdad que no se cumple para todo $x\gt 1$. Por tanto, para todo $x\gt 1$ hay un triángulo cuyos lados tienen las longitudes del enunciado.
Si crees que el enunciado contiene un error o imprecisión o bien crees que la información sobre la procedencia del problema es incorrecta, puedes notificarlo usando los siguientes botones:
Informar de error en enunciado Informar de procedencia del problema
Problema 1036
Determina el máximo valor posible de la expresión \[27abc+a\sqrt{a^2+2bc}+b\sqrt{b^2+2ac}+c\sqrt{c^2+2ab},\] siendo $a,b,c$ números reales positivos tales que $a+b+c=\frac{1}{\sqrt{3}}$.
Sin pistas
Sin soluciones
info
Si crees que el enunciado contiene un error o imprecisión o bien crees que la información sobre la procedencia del problema es incorrecta, puedes notificarlo usando los siguientes botones:
Informar de error en enunciado Informar de procedencia del problema
Problema 1013
Sea $n\geq 2$ un entero. Determinar el menor número real positivo $\gamma$ que cumple el siguiente enunciado:

Para todos los reales $x_1,x_2,\ldots,x_n\gt 0$ y $0\leq y_1,y_2,\ldots,y_n\leq\frac{1}{2}$ tales que \[x_1+x_2+\ldots+x_n=y_1+y_2+\ldots+y_n=1,\] se tiene que \[x_1x_2\cdots x_n\leq\gamma(x_1y_1+x_2y_2+\ldots+x_ny_n).\]

Sin pistas
Sin soluciones
info
Si crees que el enunciado contiene un error o imprecisión o bien crees que la información sobre la procedencia del problema es incorrecta, puedes notificarlo usando los siguientes botones:
Informar de error en enunciado Informar de procedencia del problema
José Miguel Manzano © 2010-2024. Esta página ha sido creada mediante software libre